Search found 50 matches


A and B are ruled out because they have misplaced modifier.

C is wrong because it says nobel prize is awarded to author's literature not to the author.

E is wordy as it introduces author at the end.

So, D is correct.

What is OA?

by jay2007

Thu Jul 10, 2008 8:32 am
Forum: Sentence Correction
Topic: Nobel Prize
Replies: 7
Views: 2047

Solving the first in equality i get the values of x as 3 or 4

1/5 < 1/(x+1) means, x=1,2,3,4,0
1/(x+1) < 1/2 means, x=3,4,5,..... or -2, -3, -4....

So, x should be 3 or 4

Solving the second equation i get the values of x as 3 or 4.

So, my answer is E.


What is the OA?

by jay2007

Tue Jul 08, 2008 9:36 am
Forum: Data Sufficiency
Topic: Need help with negative inequality
Replies: 9
Views: 3436

sameerbhagwatmba1,

Doesn't D change the meaning of the sentence by replacing "likely most", with "if not most"?

I would go with A.

What is the OA?

by jay2007

Tue Jul 08, 2008 9:12 am
Forum: Sentence Correction
Topic: 1000 sc question # 89
Replies: 6
Views: 4469

In 1 hour A finishes 1/x of the deck and B finishes X decks. When A and B work together in 1 hour they could finish (1/x + x) of the deck or (x^2 + 1)/x. It takes x/(x^2+1) hrs to complete one deck. After "A" has worked alone for "y" hours, remaining number of decks would be (100...

by jay2007

Mon Jun 30, 2008 9:48 pm
Forum: Problem Solving
Topic: machine rates
Replies: 1
Views: 1498

If both A and B work together they can complete the job in xy/(x+y) hours. In "y" hours B completes "1" job Therefore in xy/(x+y) hours B completes x/(x+y) job. So the remaining portion of the job is: 1-x/(x+y) which is y/(x+y). So, the answer is E. Could you please post the OA?

by jay2007

Mon Jun 30, 2008 8:07 pm
Forum: Problem Solving
Topic: Work rates - fraction of a job
Replies: 3
Views: 5178

What is the OA?

IMO, statement 2 itself is sufficient.

distance between t and r = distance between t and -s
This implies r = -s.

Isn't this enough to say that 0 is between s and -s and hence r and s?

by jay2007

Thu Oct 18, 2007 9:05 am
Forum: Data Sufficiency
Topic: number line - gmatprep
Replies: 3
Views: 1783

Think of a simplified problem: 4 persons forming 2 sub committees of 2 each. Persons: A, B, C and D 4c2 * 2c2 - Number of total combinations Comm1 comm2 A B C D C D A B --- --- First two combinations are really the same. Hence the number of possibilities is reduced by half. Hence for 6 persons formi...

by jay2007

Wed Oct 10, 2007 7:54 pm
Forum: Problem Solving
Topic: Probability - MGAMT
Replies: 6
Views: 1999

santsark is right. I also assumed that y > 36 and choose C, which is not the right answer.

OA is E.

by jay2007

Wed Oct 10, 2007 7:44 pm
Forum: Data Sufficiency
Topic: DS question from GMAT PREP - III
Replies: 4
Views: 1734

You are right. It is A.

I went little further (probably thought too much...) and chose B, which is the wrong choice.

by jay2007

Tue Oct 09, 2007 6:08 am
Forum: Critical Reasoning
Topic: GMAT Prep Critical reasoning - II
Replies: 3
Views: 2036

Thanks arocks. I made a mistake in STU. <UST and <UTS are equal. But in my equations i considered <STU and <SUT are equal.

Thanks

by jay2007

Tue Oct 09, 2007 6:03 am
Forum: Data Sufficiency
Topic: DS question from GMAT PREP
Replies: 2
Views: 1574
by jay2007

Mon Oct 08, 2007 10:12 pm
Forum: Data Sufficiency
Topic: DS question from GMAT PREP - III
Replies: 4
Views: 1734
by jay2007

Mon Oct 08, 2007 10:11 pm
Forum: Data Sufficiency
Topic: DS question from GMAT PREP - II
Replies: 1
Views: 1549

DS question from GMAT PREP

Can somebody help me in solving this problem?

by jay2007

Mon Oct 08, 2007 10:11 pm
Forum: Data Sufficiency
Topic: DS question from GMAT PREP
Replies: 2
Views: 1574

Thanks arocks. I have a question - D: If the doctors are prescribing more generic drugs instead of expensive prescription drugs, the per capita expenditure on drugs should decrease. - In this case is the assumption that generic drugs costs less than prescription drugs correct? A - The manufacturers ...

by jay2007

Mon Oct 08, 2007 10:10 pm
Forum: Critical Reasoning
Topic: GMAT Prep Critical reasoning
Replies: 4
Views: 2386

GMAT Prep Critical reasoning - II

Can somebody help me in finding the answer?

OA is E

by jay2007

Mon Oct 08, 2007 9:04 pm
Forum: Critical Reasoning
Topic: GMAT Prep Critical reasoning - II
Replies: 3
Views: 2036